13
$\begingroup$

Given $x \in [0, 1]$, we denote by $e(x)$ the complex number $e^{2 \pi i x}$.

Can we characterise the set of rationals $x$ for which the sum

$$A_N(x)\, :=\, \sum_{n = 0}^N e(2^n x)$$

remains bounded as $N \to \infty$?

Remarks:

  • We allow the bound to depend on $x$, but not $N$.

  • It can be shown that the sum is unbounded for almost every (but not every!) irrational $x$.

$\endgroup$
13
  • 4
    $\begingroup$ Nearly everyone uses the notation $e(x)$ for $e^{2\pi i x}$ if they use any notation at all. You care about the sum $\sum_{n = 0}^N e(2^n x)$ (you have the index of the sum as $i$ by mistake). $\endgroup$ Commented Oct 13, 2024 at 17:08
  • 4
    $\begingroup$ Let $q$ be the denominator of $x$. We may assume that $q$ is odd. Note that the summands are periodic with period $\varphi(q)$ (Euler totient function). So the question actually is when $\sum_{n=1}^{\varphi(q)}\alpha_x^{2^n}=0$. $\endgroup$ Commented Oct 13, 2024 at 17:09
  • 7
    $\begingroup$ If I am not mistaken, for $x$ with denominators $p^m$ with odd prime $p$ this holds iff $\nu_p(2^{p-1}-1)<m$ $\endgroup$ Commented Oct 13, 2024 at 18:58
  • 2
    $\begingroup$ @FedorPetrov I think this is right and this shows that a necessary condition is that there exists a prime $m$ dividing the denominator with multiplicity $m$ such that $v_p ( 2^{p-1}-1)<m$. (Proof: Multiply a bunch of sums with prime power denominator together to get a sum of Galois conjugates of the original sum.) But probably that's not also sufficient. $\endgroup$ Commented Oct 14, 2024 at 15:52
  • 2
    $\begingroup$ @WillSawin a slightly different sufficient condition is that the order of 2 modulo $n$ is not equal to the order of 2 modulo the product of all prime divisors of $n$ (then we may partition everything onto regular $p$-gons). I expect it to be necessary. $\endgroup$ Commented Oct 14, 2024 at 16:05

2 Answers 2

17
$\begingroup$

As Fedor Petrov expects in the comments, this is true for $x=a/b$ if and only if the order of $2$ modulo $b$ is greater than the order of $2$ modulo the product of prime factors of $b$.

To prove this, the first key observation is given by Peter Mueller in the comments: Since the sequence is eventually periodic, it is bounded if and only if the sum over a period is $0$. We can also just drop the first few terms by multiplying $x$ by a power of $2$ until $b$ is odd, in which case the sequence is literally periodic. So it is equivalent to say $$\sum_{n=0}^{m-1} e( 2^n a/b) =0 $$ where $m$ is the multiplicative order of $2$ modulo $b$.

The next key observation is from GH from MO in the comments, which is that $\sum_{n=0}^{m-1} e( 2^n a/b) $ is a Galois conjugate of $\sum_{n=0}^{m-1} e( 2^n a'/b) $ whenever $a'$ and $a$ are both relatively prime to $b$, so one of them is zero if and only if all of these are zero, i.e. if and only if $$\sum_{n=0}^{m-1} e( 2^n a'/b) =0 \textrm{ for all } a' \in (\mathbb Z/b\mathbb Z)^\times$$

Now finally I will make a contribution of my own: By Fourier analysis, we have $$\sum_{n=0}^{m-1} e( 2^n a'/b) =0 \textrm{ for all } a' \in (\mathbb Z/b\mathbb Z)^\times$$ if and only if $$ \sum_{c \in (\mathbb Z/b\mathbb Z)^\times} e( c/b) \chi(c) =0 \textrm{ for all } \chi\colon (\mathbb Z/b\mathbb Z)^\times \to \mathbb C^\times \textrm{ such that } \chi(2)=1.$$

Indeed the first sums depend on the equivalence class of $a'$ in $(\mathbb Z/b\mathbb Z)^\times/\langle 2\rangle$, and the second sums give the finite Fourier transform over this group.

Now the theory of Gauss sums tells us that $\sum_{c \in (\mathbb Z/b\mathbb Z)^\times} e( c/b) \chi(c) =0 $ if and only if $\chi$ factors through $(\mathbb Z/(b/p)\mathbb{Z})^\times$ for some $p$ with $p^2$ dividing $b$.

If the order of $2$ modulo $b$ is greater than the order of $2$ modulo the product of prime factors of $p$, raising $2$ to the order modulo the product of prime factors and then raising to a suitable product of prime factors, we can find a power of $2$ that is congruent to $1$ mod $b$ but not modulo $b/p$ for some such prime, showing that every $\chi$ admits such a factorization. Conversely, if these orders are equal, then every character of the kernel of $(\mathbb Z/ b \mathbb Z)^\times \to (\mathbb Z/r \mathbb Z)^\times$, with $r$ the product of prime factors, extends to a character of $(\mathbb Z/ b \mathbb Z)^\times$ trivial on $2$. The kernel is cyclic and extending a faithful character produces a nonzero Gauss sum.

$\endgroup$
2
  • 6
    $\begingroup$ 1. This is a nice example of virtual MO collaboration (transcending space and time)! The credit is all yours, of course. 2. Your argument reminds me of certain nonvanishing results for central $L$-values (e.g. by Rohrlich) where the values are Galois conjugates, so it suffices to show that their sum is nonzero. 3. I fixed some typos. $\endgroup$ Commented Oct 15, 2024 at 0:44
  • 2
    $\begingroup$ @GHfromMO Collaboration transcending time, space and even language has been a big theme in my life recently... anyway really nicely done to everyone. $\endgroup$ Commented Oct 15, 2024 at 2:41
1
$\begingroup$

Not a full answer, but here is a potentially useful observation - let $x$ be a real number, rational or otherwise. By writing $x$ in binary, truncating to a certain number of binary digits to the right of the decimal point, and then counting the error terms, we can show that

$$A_N (x) \leq o(1) + \inf_{k \geq 2 \log_2 N} \sum_{n = 1}^{N} e(T_{k}(2^n x)).$$

where $T_m$ denotes the truncation to $m$ binary digits operator, and the $o(1)$ denotes an error term independent of $x$ that goes to $0$ in absolute value as $N \to \infty$.

This formula allows to get very precise estimates on the sum. Indeed, since $x \to 2^n x$ is just the shift map in binary, we can use this formula to show that in fact, for the rational number $\frac{10}{99} = 0.10101010\dots$, the sum $A_N (\frac{10}{99})$ is in fact bounded as $N \to \infty$ (see here for proof!), and we may even obtain the exact explicit bound if desired.

The formula may also be useful for checking boundedness on irrationals.

$\endgroup$

You must log in to answer this question.

Start asking to get answers

Find the answer to your question by asking.

Ask question

Explore related questions

See similar questions with these tags.